השינוי האחרון נעשה בֹ־16 בפברואר 2012 ב־12:07

שיחת משתמש:מני ש.

חזרה לדף "מני ש.".

פה, ב2ג, מחקת את ההוכחה שלי רק כדי לכתוב ניסוח אחר של אותו הדבר...

למה?

היתה טעות בהוכחה המקורית. היה רשום שם בין השאר \log(\frac{1}{n})\geq \frac{1}{n} וזה לא נכון שהרי\log(\frac{1}{n}) שלילי לכל n\geq 2